AMC 10 A
AMC 10 B
AMC 12 A(all q's x2)
AMC 12 B(all q's x2)
100

AMC 10A 2021 Problem 1

What is the value of

100

AMC 10B 2017 Problem 3

Real numbers , , and  satisfy the inequalities , , and . Which of the following numbers is necessarily positive?

(E)y+z

Notice that  must be positive because . Therefore the answer is .

The other choices:

 As  grows closer to ,  decreases and thus becomes less than .

  can be as small as possible (), so  grows close to  as  approaches .

 For all , , and thus it is always negative.

 The same logic as above, but when  this time.

https://artofproblemsolving.com/wiki/index.php/2017_AMC_10B_Problems/Problem_3

100

AMC 12A 2021 Problem 1

What is the value of

(B)50

We evaluate the given expression to get that

100

AMC 12B 2004 Problem 1

At each basketball practice last week, Jenny made twice as many free throws as she made at the previous practice. At her fifth practice she made 48 free throws. How many free throws did she make at the first practice?

(A)3

Each day Jenny makes half as many free throws as she does at the next practice. Hence on the fourth day she made  free throws, on the third , on the second , and on the first .

Because there are five days, or four transformations between days (day 1  day 2  day 3  day 4  day 5), she makes

200

AMC 10A 2021 Problem 8

When a student multiplied the number  by the repeating decimal,  where  and  are digits, he did not notice the notation and just multiplied  times  Later he found that his answer is  less than the correct answer. What is the -digit number

200

AMC 10B 2017 Problem 7

Samia set off on her bicycle to visit her friend, traveling at an average speed of  kilometers per hour. When she had gone half the distance to her friend's house, a tire went flat, and she walked the rest of the way at  kilometers per hour. In all it took her  minutes to reach her friend's house. In kilometers rounded to the nearest tenth, how far did Samia walk?

(C)2.8

Let's call the distance that Samia had to travel in total as , so that we can avoid fractions. We know that the length of the bike ride and how far she walked are equal, so they are both , or .She bikes at a rate of  kph, so she travels the distance she bikes in  hours. She walks at a rate of  kph, so she travels the distance she walks in  hours.The total time is . This is equal to  of an hour. Solving for , we have:Since  is the distance of how far Samia traveled by both walking and biking, and we want to know how far Samia walked to the nearest tenth, we have that Samia walked about .

https://artofproblemsolving.com/wiki/index.php/2017_AMC_10B_Problems/Problem_7

200

AMC 12B 2021 Problem 6

A deck of cards has only red cards and black cards. The probability of a randomly chosen card being red is . When  black cards are added to the deck, the probability of choosing red becomes . How many cards were in the deck originally?

(C)12

If the probability of choosing a red card is , the red and black cards are in ratio . This means at the beginning there are  red cards and  black cards.

After  black cards are added, there are  black cards. This time, the probability of choosing a red card is  so the ratio of red to black cards is . This means in the new deck the number of black cards is also  for the same  red cards.

So,  and  meaning there are  red cards in the deck at the start and  black cards.

So, the answer is .

200

AMC 12B 2004 Problem 5

On a trip from the United States to Canada, Isabella took  U.S. dollars. At the border she exchanged them all, receiving  Canadian dollars for every  U.S. dollars. After spending  Canadian dollars, she had  Canadian dollars left. What is the sum of the digits of ?

(A)5

Isabella had  Canadian dollars. Setting up an equation we get , which solves to , and the sum of digits of  is .

300

AMC 10A 2011 Problem 15

Roy bought a new battery-gasoline hybrid car. On a trip the car ran exclusively on its battery for the first  miles, then ran exclusively on gasoline for the rest of the trip, using gasoline at a rate of  gallons per mile. On the whole trip he averaged  miles per gallon. How long was the trip in miles?

(C) 440

We know that . Let  be the distance the car traveled during the time it ran on gasoline, then the amount of gas used is . The total distance traveled is , so we get . Solving this equation, we get , so the total distance is .

300

AMC 10B 2015 Problem 12

For how many integers  is the point  inside or on the circle of radius  centered at ?

(A)11

The equation of the circle is . Plugging in the given conditions we have . Expanding gives: , which simplifies to  and therefore  and . So  ranges from  to , for a total of  integer values.

Note by Williamgolly: Alternatively, draw out the circle and see that these points must be on the line

https://artofproblemsolving.com/wiki/index.php/2015_AMC_10B_Problems/Problem_12

300

AMC 12A 2021 Problem

Of the following complex numbers , which one has the property that  has the greatest real part?

First,  is ,  is ,  is .

Taking the real part of the th power of each we have:

, whose real part is

Thus, the answer is .

300

AMC 12B 2003 Problem 21

An object moves  cm in a straight line from  to , turns at an angle , measured in radians and chosen at random from the interval , and moves  cm in a straight line to . What is the probability that ?

(D)1/3


It follows that , and the probability is .

400

AMC 10A 2011 Problem 19

In 1991 the population of a town was a perfect square. Ten years later, after an increase of 150 people, the population was 9 more than a perfect square. Now, in 2011, with an increase of another 150 people, the population is once again a perfect square. Which of the following is closest to the percent growth of the town's population during this twenty-year period?

(E) 62%

Let the population of the town in  be . Let the population in  be . It follows that . Rearrange this equation to get . Since  and  are both positive integers with ,  and  also must be, and thus, they are both factors of . We have two choices for pairs of factors of :  and , and  and . Assuming the former pair, since  must be less than ,  and . Solve to get . Since  is not a perfect square, this is not the correct pair. Solve for the other pair to get . This time, . This is the correct pair. Now, we find the percent increase from  to . Since the increase is , the percent increase is .

https://artofproblemsolving.com/wiki/index.php/2011_AMC_10A_Problems/Problem_19


400

AMC 10B 2006 Problem 19

Let  be a sequence for which  , , and  for each positive integer . What is ?

(E)3

Looking at the first few terms of the sequence:

Clearly, the sequence repeats every 6 terms.

Since ,


400

AMC 12A 2009 Problem 20

Convex quadrilateral  has  and . Diagonals  and  intersect at , , and  and  have equal areas. What is ?

(E)6

Let  denote the area of triangle . , so . Since triangles  and  share a base, they also have the same height and thus  and  with a ratio of . , so

400

AMC 12B 2004 Problem 12

In the sequence , , ,  , each term after the third is found by subtracting the previous term from the sum of the two terms that precede that term. For example, the fourth term is . What is the  term in this sequence?

(C)0

We already know that , , , and . Let's compute the next few terms to get the idea how the sequence behaves. We get , , , and so on.

We can now discover the following pattern:  and . This is easily proved by induction. It follows that .

500

AMC 10A 2007 Problem 25

For each positive integer , let  denote the sum of the digits of  For how many values of  is

(D)4

For the sake of notation let . Obviously . Then the maximum value of  is when , and the sum becomes . So the minimum bound is . We do casework upon the tens digit:

Case 1: . Easy to directly disprove.

Case 2: . , and  if  and  otherwise.

Subcase a: . This exceeds our bounds, so no solution here.Subcase b: . First solution.

Case 3: . , and  if  and  otherwise.

Subcase a: . Second solution.Subcase b: . Third solution.

Case 4: . But , and  clearly sum to .

Case 5: . So  and  (recall that ), and . Fourth solution.

In total we have  solutions, which are  and .

https://artofproblemsolving.com/wiki/index.php/2007_AMC_12A_Problems/Problem_22 

500

AMC 10B 2006 Problem 25

Mr. Jones has eight children of different ages. On a family trip his oldest child, who is 9, spots a license plate with a 4-digit number in which each of two digits appears two times. "Look, daddy!" she exclaims. "That number is evenly divisible by the age of each of us kids!" "That's right," replies Mr. Jones, "and the last two digits just happen to be my age." Which of the following is not the age of one of Mr. Jones's children?

(B)5

Let  be the set of the ages of Mr. Jones' children (in other words  if Mr. Jones has a child who is  years old). Then  and . Let  be the positive integer seen on the license plate. Since at least one of  or  is contained in , we have .

We would like to prove that , so for the sake of contradiction, let us suppose that . Then  so the units digit of  is . Since the number has two distinct digits, each appearing twice, another digit of  must be . Since Mr. Jones can't be  years old, the last two digits can't be . Therefore  must be of the form , where  is a digit. Since  is divisible by , the sum of the digits of  must be divisible by  (see Divisibility rules for 9). Hence  which implies . But  is not divisible by , contradiction. So  and  is not the age of one of Mr. Jones' kids.

(We might like to check that there does, indeed, exist such a positive integer . If  is not an age of one of Mr. Jones' kids, then the license plate's number must be a multiple of . Since  and  is the only  digit multiple of  that fits all the conditions of the license plate's number, the license plate's number is .)

https://artofproblemsolving.com/wiki/index.php/2006_AMC_10B_Problems/Problem_25

500

AMC 12A 2009 Problem 25

The first two terms of a sequence are  and . For ,

What is ?


(A)0

Consider another sequence  such that , and .

The given recurrence becomes

It follows that . Since , all terms in the sequence  will be a multiple of .

Now consider another sequence  such that , and . The sequence  satisfies .

As the number of possible consecutive two terms is finite, we know that the sequence  is periodic. Write out the first few terms of the sequence until it starts to repeat.

Note that  and . Thus  has a period of : .

It follows that  and . Thus

Our answer is .

500

AMC 12B 2003 Problem 25

Three points are chosen randomly and independently on a circle. What is the probability that all three pairwise distance between the points are less than the radius of the circle?

(D)1/12

The first point anywhere on the circle, because it doesn't matter where it is chosen.

The next point must lie within  degrees of arc on either side, a total of  degrees possible, giving a total  chance. The last point must lie within  degrees of both.

The minimum area of freedom we have to place the third point is a  degrees arc(if the first two are  degrees apart), with a  probability. The maximum amount of freedom we have to place the third point is a  degree arc(if the first two are the same point), with a  probability.

As the second point moves farther away from the first point, up to a maximum of  degrees, the probability changes linearly (every degree it moves, adds one degree to where the third could be).

Therefore, we can average probabilities at each end to find  to find the average probability we can place the third point based on a varying second point.

Therefore the total probability is  or

M
e
n
u